WBR0943
Author | PageAuthor::William J Gibson |
---|---|
Exam Type | ExamType::USMLE Step 1 |
Main Category | MainCategory::Pharmacology |
Sub Category | SubCategory::Oncology, SubCategory::Renal |
Prompt | [[Prompt::A 72-year-old man with end-stage Chronic Lymphocytic Leukemia is administered an experimental treatment with tumor-targeting T-cells. Approximately two weeks after receiving the injection, the patient develops a fever of 104 F with shaking and chills. Laboratory studies reveal a thousand-fold increase in IL6 along with the following:
Sodium: 140 mEq/L Potassium: 5.5 mm/L Phosphate: 5.0 mg/dL Calcium: 8.0 mg/dL Uric acid: 16.0 mg/dL (Normal 2.0 – 8.0) Lactic Acid: 5.0 mEq/L (Normal 0.7-2.1) Which of the following drugs should be avoided in this patient?]] |
Answer A | AnswerA::Hydrochlorothiazide |
Answer A Explanation | AnswerAExp:: |
Answer B | AnswerB::Furosemide |
Answer B Explanation | AnswerBExp:: |
Answer C | AnswerC::Corticosteroids |
Answer C Explanation | AnswerCExp:: |
Answer D | AnswerD::Allopurinol |
Answer D Explanation | AnswerDExp:: |
Answer E | AnswerE::Acetazolamide |
Answer E Explanation | AnswerEExp:: |
Right Answer | RightAnswer:: |
Explanation | [[Explanation:: Educational Objective: |
Approved | Approved::No |
Keyword | |
Linked Question | Linked:: |
Order in Linked Questions | LinkedOrder:: |